0 Daumen
1,3k Aufrufe

Soll per Widerspruchsbeweis gelöst werden.

n∈N

Mein Versuch:

Angenommen es gibt nur endlich viele n für die Wurzel(n) irrational ist.

Dann gibt es ein größtes nmax  für das Wurzel(nmax) irrational ist.

Wähle jetzt nK = 4*nmax .

Dann lässt sich die Wurzel daraus schreiben als:

wurzel(nK) = p/q

= wurzel(4*nmax)

⇔ wurzel(4) * wurzel(nmax) = p/q

⇔ wurzel(nmax) = p/(2q)

Was ein Widerspruch ist zur Annahme, dass wurzel(nmax) irrational ist.


So richtig?

Avatar von

p,q sollen dabei natürlich aus N sein..

Und? Ueberzeugt Dich der Beweis selber? Wie bist Du drauf gekommen? Wo siehst Du eventuell Probleme? Oder hast Du ihn bloss abgeschrieben und nicht recht verstanden?

Bonusaufgabe: Gib unendlich viele n mit der gewuenschten Eigenschaft explizit an!

nk = 4 * nmax

Ich denke mal k ∈ N... aber wieso 4 * nmax ?

Ich habe nicht die geringste Ahnung, was das K in nK soll. Das musst Du ||||||||||||||||| fragen.

Ich habe nicht die geringste Ahnung, was das K in nK soll. Das musst Du ||||||||||||||||| fragen.

Ich denke mal das ist nur ein Bezeichner für die neue größte Zahl n... man hätte auch nneu schreiben können.

So viel ist sicher. Warum denkst Du dann oben mal k ∈ N ?

So viel ist sicher. Warum denkst Du dann oben mal k ∈ N ?

Da hab ich falsch geguckt... ich dachte das wäre ein Zähler für n.

1 Antwort

+1 Daumen
 
Beste Antwort

Hallo dein Beweis ist ok.

da es unendlich viele Primzahlen gibt würde das auch ausreichen. Gruß lul

Avatar von 106 k 🚀

Ein anderes Problem?

Stell deine Frage

Willkommen bei der Mathelounge! Stell deine Frage einfach und kostenlos

x
Made by a lovely community